Головоломка о теореме о расходимости

Что-то меня озадачивает в теореме о дивергенции. Обычно теорему о расходимости записывают как

(3.22) М д 4 Икс г мю в мю "=" М д Σ мю в мю
для некоторого векторного поля в мю , где д Σ мю – направленный элемент поверхности. См., например, уравнение ( 3,22 ) в «Инструментарии релятивиста» Эрика Пуассона. Я подумывал о том, чтобы превратить лагранжиан Максвелла в граничный член, предполагая, что я нахожусь на раковине. Для этого у нас есть
М д 4 Икс г Ф мю ν Ф мю ν "=" 2 М д 4 Икс г Ф мю ν мю А ν "=" 2 М д 4 Икс г мю ( Ф мю ν А ν ) 2 М д 4 Икс г мю Ф мю ν А ν
где я просто использовал симметрию символов Кристоффеля и антисимметрию тензора Фарадея. Теперь, используя уравнение движения
мю Ф мю ν "=" 0
и теорему о расходимости, я должен быть в состоянии написать
М д 4 Икс г Ф мю ν Ф мю ν "=" 2 М д Σ мю Ф мю ν А ν .
Теперь мы можем рассмотреть магнитное решение, заданное формулой
А "=" Вопрос ( 1 потому что θ ) д ф
Ф "=" д А "=" Вопрос грех θ д θ д ф .

Учитывая это решение, мы имеем

Ф мю ν Ф мю ν "=" 2 Вопрос 2 р 4
предполагая метрику вида
д с 2 "=" ф ( р ) д т 2 + 1 ф ( р ) д р 2 + р 2 ( д θ 2 + грех 2 θ д ф 2 ) .
На самом деле я имею дело с черными дырами Рейснера-Нордстрема с магнитным зарядом, но я думаю, что мое замешательство имеет более общее значение. Меня смущает то, что если я возьму какую-то границу постоянной р , или просто некоторая граница, нормаль которой выглядит как
н мю "=" α д т + β д р ,
затем правая сторона
М д 4 Икс г Ф мю ν Ф мю ν "=" 2 М д Σ мю Ф мю ν А ν .
кажется, тривиально дает ноль, потому что Ф имеет только угловые компоненты. Однако я думаю, что очень легко придумать область пространства-времени. М такое, что левая часть не обращается в нуль. Что я сделал не так в этом рассуждении? Я думал, что это может быть связано с гладкостью векторного поля. в мю "=" Ф мю ν А ν в котором мы применяем теорему Стокса, но поскольку я видел изложенную выше теорему о дивергенции без каких-либо предположений о векторном поле, я не уверен, что проблема в этом. Векторное поле дает
в "=" Вопрос 2 р 4 грех θ ( 1 потому что θ ) θ ,
Это можно увидеть в имеет сингулярность в θ "=" π . Это может быть артефакт координат, но легко увидеть, что г мю ν в мю в ν также имеет эту особую точку. По этой причине я думаю, что может быть проблематично применить теорему о расходимости к в . Я правильно понял проблему?

EDIT1: я рассматриваю регион М пространства-времени, не содержащего начала. Поскольку я думаю об этом в контексте черной дыры, я не хочу включать сингулярность р "=" 0 в патче, поверх которого я интегрируюсь. В качестве примера этой процедуры см. уравнение ( 5.15 ) https://arxiv.org/abs/1606.08307 , где предполагается электрическое решение. Теперь, если мы примем электрическое решение, мы получим

А "=" ( Вопрос р + Вопрос р ) д т
Ф "=" д А "=" Вопрос 2 р 2 д р д т
где р + является горизонтом событий. Следовательно
Ф мю ν Ф мю ν "=" 2 Вопрос 2 р 4 ,
что означает массовую интеграцию на М просто меняется на знак относительно магнитного решения. Это связано с электромагнитной двойственностью. Однако в этом случае использование теоремы Стокса приводит к ненулевому граничному вкладу, как, я думаю, и должно быть. Теперь моя загадка заключается в том, почему в одном случае мы можем использовать Стокса, а в другом, кажется, нет. Оказывается, в этом случае векторное поле, в котором мы применяем Стокса, равно
в "=" Вопрос р 2 ( 1 р 1 р + ) р
который кажется гладким везде, если мы посмотрим на г ( в , в ) , используя хорошую систему координат.

РЕДАКТИРОВАТЬ 2: Согласно «Введению в гладкие многообразия» Джона М. Ли, векторное поле должно быть гладким. Но если это так, то когда мы используем вариационный принцип для вывода уравнений Максвелла, мы предполагаем, что Ф мю ν дельта А ν является гладким, чтобы превратить его в граничный член, который равен нулю. Конечно тут мы не в ладах, но мне все же кажется странным, что Ф мю ν А мю оказывается негладким для чисто магнитного решения. Есть ли у вас какие-либо идеи?

Обратите внимание, что магнитное решение имеет точечный источник в начале координат, поэтому мю Ф мю ν не обращается в нуль, а пропорциональна дельта ( Икс ) . На самом деле, вы, вероятно, могли бы повторить свои рассуждения для электрического точечного источника и в плоском пространстве-времени, чтобы прийти к той же самой проблеме, но исключив все тангенциальные вещи.
Но я могу рассмотреть регион М пространства-времени, не содержащего начала. Поскольку я думаю об этом в контексте черной дыры, я не хочу включать сингулярность р "=" 0 в патче, поверх которого я интегрируюсь. Тогда он исчезает, верно?

Ответы (2)

Это происходит не из-за поведения только в начале, а из-за плохо определенного поведения А вдоль линии θ "=" π . д ф там не имеет смысла и коэффициент при нем не обращается в нуль. Интегрируйте, избегая этой полулинии, и ваш интеграл будет работать.

В плоском пространстве я буду интегрировать θ от 0 к θ м , р а к р б , и вообще ф и интервал времени Т . Нормаль к граничной поверхности находится в р направление везде, кроме поверхности в θ "=" θ м что, конечно, нормально для θ направление. Это точно в сторону Ф мю ν А мю , и это будет единственный вклад в поверхностный интеграл.

Ваш вектор в мю "=" Ф мю ν А ν находится в θ направление и имеет величину

| в | "=" Вопрос 2 р 3 грех θ ( 1 потому что θ )
Теперь поверхностный интеграл равен
Т р а р б | в | 2 π р грех θ м д р "=" 2 π Т Вопрос 2 ( 1 потому что θ м ) р а р б д р р 2 4 π Т Вопрос 2 р а р б д р р 2

Где в последней строке я могу взять θ м "=" π теперь, когда А поле исчезло. Это и есть объемный интеграл Вопрос 2 / р 4 . Мы видим площадь поверхности 4 π временной интервал Т и фактор р 2 исходя из меры интегрирования.

Большое спасибо! Сначала я не понял, что вы звоните | в | но теперь я понимаю, что это просто г ( в , в ) . Так что да, это имеет смысл, спасибо.

Если убрать механизм искривления пространства-времени, то вы только что пришли к обычной тонкости, связанной с магнитными монополями. То есть, если предположить Ф "=" д А то у вас автоматически д Ф "=" 0 , который включает в себя закон Гаусса для магнетизма Б "=" 0 запрещающие магнитные монополи. Переход к искривленному пространству-времени делает эти уравнения немного причудливее, но на самом деле не меняет логику.

Чтобы разрешить монополии, мы должны либо:

  • перейти к формализму расслоения, описывая А как связь на U ( 1 ) расслоить ваше пространство-время, или
  • объяснить «струну Дирака», сингулярность в А это неизбежно появляется, если вы не используете пучки волокон, что в данном случае происходит при θ "=" π .

Какой бы вариант вы ни выбрали, техническое решение будет одинаковым. Если вы используете пакеты, вам придется покрыть свой р "=" константа поверхность с двумя патчами, и вы получите дополнительные термины от перекрытия. Если вы используете струну Дирака, вы должны вырезать часть поверхности, через которую проходит струна Дирака, и это дает границу, которая дает вклад в правую часть теоремы о расходимости.

Большое спасибо кнжоу! Строковая процедура Дирака, на которую вы ссылаетесь, в основном явная с помощью @octonion, верно? У вас есть какая-нибудь ссылка, которая объясняет, как это будет сделано с использованием пучков волокон? Еще раз спасибо!
@blackhole1511 Да, это то же самое, что и они. Для связок есть одно хорошее объяснение в « Классических решениях в квантовой теории поля» Вайнберга .